我正在尝试使用eqnarray
考试类别来解答问题:
\begin{questions}
\printanswers
\question What is $10 - 3$?
\begin{solution}
\begin{eqnarray}
10 - 3 &= 10 + (-3) \\
&= 7 + 3 + (-3 )\\
&= 7 + 0\\
&= 7
\end{eqnarray}
\end{solution}
\end{questions}
结果格式不正确:
有解决方法吗?
答案1
如果您不想要编号,这里有一个“或类似”的方法。此外,它可以移动到任何水平位置。我通过用\hfil
s 包围它来使其居中:
\documentclass{exam}
\usepackage{tabstackengine}
\stackMath
\begin{document}
\begin{questions}
\printanswers
\question What is $10 - 3$?
\begin{solution}
\par\hfil\alignCenterstack{%
10 - 3 =& 10 + (-3) \\
=& 7 + 3 + (-3 )\\
=& 7 + 0\\
=& 7
}\hfil
\end{solution}
\end{questions}
\end{document}
使用这种方法的一个优点是它将方程式放在 LaTeX“框”中,因此可以在页面上以无限的方式移动和操作它们。例如,在这里,您可以布置两个并行解决方案策略:
\documentclass{exam}
\usepackage{tabstackengine}
\stackMath
\begin{document}
\begin{questions}
\printanswers
\question What is $10 - 3$?
\begin{solution}
\par\hfil\alignCenterstack{%
10 - 3 =& 10 + (-3) \\
=& 7 + 3 + (-3 )\\
=& 7 + 0\\
=& 7
}\hfil\alignCenterstack{%
10 - 3 =& 10 + (-3) \\
=& 10 + (7 - 10)\\
=& 10 +(- 10) + 7\\
=& 7
}\hfil
\end{solution}
\end{questions}
\end{document}
甚至添加评论:
\documentclass{exam}
\usepackage{tabstackengine}
\usepackage{xcolor}
\usepackage{graphicx}
\stackMath
\begin{document}
\begin{questions}
\printanswers
\question What is $10 - 3$?
\begin{solution}
\par\hfil\alignCenterstack{%
10 - 3 =& 10 + (-3) \\
=& 7 + 3 + (-3 )\\
=& 7 + 0\\
=& 7
}\hfil\stackinset{r}{3.7ex}{t}{-3.2ex}%
{\color{red}\stackon{\scalebox{8}{$\times$}}{\mathrm{Don't~do~this!}}}%
{\alignCenterstack{%
10 - 3 =& 10 + (-3) \\
=& 10 + (7 - 10)\\
=& 10 +(- 10) + 7\\
=& 7
}}\hfil
\end{solution}
\end{questions}
\end{document}
答案2
最好的方法是使用amsmath
及其环境align
。请参阅以下 MWE:
\documentclass{exam}
\usepackage{amsmath}
\begin{document}
\begin{questions}
\printanswers
\question What is $10 - 3$?
\begin{solution}
\begin{align}
10 - 3 &= 10 + (-3) \\
&= 7 + 3 + (-3 )\\
&= 7 + 0\\
&= 7
\end{align}
\end{solution}
\end{questions}
\end{document}
请记住,允许使用错误的语法,例如eqnarray
。有关更多提示,请参阅禁忌。